The height and radius of a cone are each multiplied by 11. What effect does this have on the volume of the cone?

The volume of the cone is multiplied by (Blank 1).

Answers

Answer 1

The volume of the new cone would be 1331 times more than the volume of the original cone. Thus, The volume of the original cone is multiplied by 1331

The volume of the cone can be calculated by the given formula:

                       The volume of cone = [tex]\frac{1}{3} \pi r^{2} h[/tex]

where r = radius of the cone

           h = height of the cone

Solution:

The volume of the original cone= .......1

It is given that the height and radius of a cone are multiplied by 11.

then volume

= [tex]\frac{1}{3} \pi (11r)^{2} (11h)[/tex]

= [tex]\frac{1}{3} \pi (121r^{2}) (11h)[/tex] .........2

The change in volume of the cone after multiplying 11 to r and h would be

dividing equation 2 by equation 1

=  [tex]\frac{1}{3} \pi (121r^{2}) (11h)[/tex]/[tex]\frac{1}{3} \pi r^{2} h[/tex]

= 121*11

= 1331

Thus, The volume of the original cone is multiplied by 1331

Learn more about the volume of the cone:

https://brainly.com/question/1315822


Related Questions

What is the length of AC?

Answers

Answer:

B) 4 miles

Step-by-step explanation:

the big triangle CAB and small triangle CED are simular (given)

this means the ratio between their corresponding sides will be the same.

find corresponding sides with alternate interior angle theorem

m CDE= m CBA

vertical angles theorem

m ACB = m DCE

therefore, AC corresponds to CE

find ratio of sides

7 miles = 36960 feet

36960/ corresponding side 21 ft

36960/ 21= 1780 ft

multiply by 1780 to get from small side to big side

small side CE = 12 ft

12*1780

=21120 ft =AC (in ft)

convert to miles (ft to miles is ft/5280)

21120/5280

=4 miles

hope this helps

The perimeter of a rectangle is 74 inches. If the length is five more than the width, what are the rectangle's measurements?

Answers

Answer:

P=2(74)+2(5)

148+10=158

Answer my question

im being timed

Answers

Answer:

the third one

Step-by-step explanation:

what is |x-4| if x>4

Answers

Answer:

x-4

Step-by-step explanation:

If x>4, |x-4|>0 which means |x-4|=x-4

X = [?]°
Dr.
X Х
As
140°
B
C

Answers

Answer:

40°

Step-by-step explanation:

x+140=180°(angle on a straight line)

x=180-140

x=40°

∠DBA and ∠DBC are making linear pair.

Linear pair of angles are formed when two lines intersect each other at a single point.

The sum of angles of a linear pair is always equal to 180°

[tex] \rm \large\longrightarrow \: \: x \: + \: 140 \degree \: = \: 180 \degree[/tex]

[tex]\rm \large\longrightarrow \: \: x \: = \: 180 \degree \: - \: 140 \degree \: [/tex]

[tex]\rm \large\longrightarrow \: \: x \: = \: 40 \degree[/tex]

What is 15.4624 divided by 0.32

Answers

Answer:

48.32

Step-by-step explanation:

What is the value of x in the equation 1/5x - 2/3y
=
30, when y = 15?

Answers

Answer:

x = 200

Step-by-step explanation:

Plug in 15 as y into the equation and solve for x:

1/5x - 2/3y = 30

1/5x - 2/3(15) = 30

1/5x - 10 = 30

1/5x = 40

x = 200

So, when y = 15, x = 200

Find the Measure of one interior angle for each polygon

Answers

Answer:

5 corners : 108 degrees

6 corners : 120 degrees

Step-by-step explanation:

there are (at least) 2 different views to get the result :

officially (usually the teachers' preferred method) you consider a polygon as a combination of non-overlapping triangles. a polygon with n corners or edges we can split into n-2 such triangles.

each triangle has an angle sum of 180 degree.

so, the polygon angle sum is (n-2)×180 degrees.

and each (internal) angle is then (n-2)×180/n

n = 5 : (5-2)×180/5 = 3×36 = 108 degrees

n = 6 : (6-2)×180/6 = 4×30 = 120 degrees

the second approach (I prefer) goes after the external angles of the polygon.

the sum of all external angles in any polygon is 360 degrees (a full circle).

for n corners/edges each external angle is 360/n.

and the internal angle is then the complement to 180 degrees = 180 - 360/n

n = 5 : 180 - 360/5 = 180 - 72 = 108 degrees

n = 6 : 180 - 360/6 = 180 - 60 = 120 degrees

help pls pls pls pls pls pls

Answers

Answer Q=(9,-7). R=(9,-1). S=(5,-4)

Find the number of all the 2-digit numbers satisfying the
following congruences x = 3(mod7), x = 2(mod5).
OLEASE HELP

Answers

Use the Chinese remainder theorem.

Start with x = 3×5 + 2×7 = 15 + 14 = 29. Now,

• 29 ≡ 15 ≡ 1 (mod7)

• 29 ≡ 14 ≡ 4 (mod5)

Adjust for this by multiplying the first term in x by 3, and the second term by 3 (because 4×3 ≡ 12 ≡ 2 (mod5)).

So now x = 3×5×3 + 2×7×3 = 45 + 42 = 87, and

• 87 ≡ 45 ≡ 3 (mod7)

• 87 ≡ 42 ≡ 2 (mod5)

The CRT then says that x ≡ 87 (mod(7×5)) ≡ 87 (mod35), which is to say any number x = 87 + 35n satisfies both congruences (where n is any integer).

So there are 3 possible 2-digit numbers that work: {17, 52, 87}.

To confirm:

• 17 ≡ 15 + 2 ≡ 2 (mod5) and 17 ≡ 14 + 3 ≡ 3 (mod7)

• 52 ≡ 50 + 2 ≡ 2 (mod5) and 52 ≡ 49 + 3 ≡ 3 (mod7)

• 87 ≡ 85 + 2 ≡ 2 (mod5) and 87 ≡ 84 + 3 ≡ 3 (mod7)

Find the measure of b.
please help!

Answers

Answer:  40

=======================================================

Explanation:

The inscribed angle 20 degrees doubles to 2*20 = 40 which is the measure of the central angle, and the arc in which the inscribed angle subtends (or cuts off). This is due to the aptly named inscribed angle theorem.

------------

A slightly longer alternative path would be to do this:

The triangle with interior angles 20 and c is isosceles. Note how the missing angle up top is one of the congruent base angles, so the missing angle is 20 degrees. That means angle c is...

20+20+c = 180

40+c = 180

c = 180-40

c = 140

Then angle b is supplementary to this

b+c = 180

b+140 = 180

b = 180-140

b = 40

This path leads to the same answer. It's slightly longer, but it's a path you can take if you aren't familiar with the inscribed angle theorem.

In fact, this line of thinking is effectively how the inscribed angle theorem is proved as shown in the diagram below.

Laura brought some fish; salt water ($2 each) and fresh water ($1 each). If she has 15 fish - how many of each did she buy?

Answers

Answer:

are you sure there is no further info

Step-by-step explanation:

I need help right now!!!ASAP

Answers

Answer:

Points Q, J and M are not collinear

Step-by-step explanation:

Three points cannot be collinear if they are not coplanar (on the same plane), and M is on a different plane than Q. J is located on both planes.

pls explain me i will make u as brainlist​

Answers

Answer:

hope this help you

have a great day

an obtuse angle is an angle which is _______

Answers

Answer:

an obtuse angle is an angle that is more than 90 degrees

Step-by-step explanation:

A point is chosen randomly on KN. Identify the probability that the point is on KL or MN.

Answers

Answer:

11/18

Step-by-step explanation:

KL + MN = 11

Total = 18

Probability of KL or MN = 11/18

HELP ASAP KHAN NEED HELP NOW ITS KHAN

Answers

Answer:

[tex]{ \bf{ \green{line \: B}}}[/tex]

It's a direct proportionality

Any help would be nice

Answers

Answer:

11. angles on a straight line add up to 180

so 180 -133 equal to b.b=47

12.Vertical opposite angles are equal.

therefore b=50

-2z² + 4z + 2z² = ?
Anyone know this? ​

Answers

Answer:

4

Step-by-step explanation:

Let's simplify step-by-step.

−2z2+4z+2z2

Combine Like Terms:

=−2z2+4z+2z2

=(−2z2+2z2)+(4z)

=4z

Answer:

4z

Hope this helps <3 Need thanks?

Comment /hearthelp

[tex]\\ \sf\longmapsto -2z^2+4z+2z^2[/tex]

Combine like. variables

[tex]\\ \sf\longmapsto -2z^2+2z^2+4z[/tex]

[tex]\\ \sf\longmapsto (-2+2)z^2+4z[/tex]

[tex]\\ \sf\longmapsto 0z^2+4z[/tex]

[tex]\\ \sf\longmapsto 4z[/tex]

Help me, please
I really need

Answers

Answer:

answrr

yes ok

Step-by-step explanation:

4+4-4#_44

You and six friends play on a basketball team. A sponsor paid $100 for the league fee, x dollars for each player’s T-shirt, and $68.25 for trophies. Write an expression for the total amount paid by the sponsor

Answers

Answer:

Total amount paid by the sponsor = 175 + 6d

Step-by-step explanation:

You and 5 friends = 6 people

Cost of renting a bus = $75

Team entry fee = $100

Cost of each student t shirts = $d

Cost of 6 student t shirts = $d × 6= $6d

Write an expression for the total amount the sponsor paid.

Total amount paid by the sponsor = Cost of renting a bus + Team entry fee + Cost of 6 student t shirts

= $75 + $100 + $6d

= $175 + $6d

Total amount paid by the sponsor = 175 + 6d

Where,

d = cost of each student t shirts

Debt is a result of spending more than you make.
True
False

Answers

The answer is false.

Write equations for the vertical and horizontal lines passing through the point (5,-9).

Answers

Answer:

the vertical line is:

x = 5

The horizontal line is:

y = -9

Step-by-step explanation:

A vertical line has a fixed x-value, while a horizontal line has a fixed y-value.

Then we can write a vertical line as:

x = a

and a horizontal line as

y = b

Then, if we want a vertical line that passes through the point (5, -9), remember that the x-vale will be fixed, then we fix the x-value at the same x-value of the point, which we know that is 5, then the vertical line that passes through the point (5, -9) is:

x = 5

While the horizontal line that passes through the point (5, -9) will be a line with the y-value fixed at the y-value of the point, which we know is -9

Then the horizontal line is:

y = -9

8 ÷ -2 · 42 + 9 i need help please

Answers

I have to write 20 characters but is is 43
The answer is 43! Have a nice day!

The sum of first three terms of an arithmetic series is 21. If the sum of the first two terms is subtracted from the third term then it would be 9. find the three terms of the series. The first and last bra​

Answers

Answer:

The terms are - 1, 7 and 15.

Step-by-step explanation:

Let the terms be a-d, a, a+d.

ATQ, a-d+a+a+d=21, a=7 and a+d-(a-d+a)=9. 2d-a=9, d=8. The terms are - 1, 7 and 15.

The first three terms will be :   -1, 7, 15

We have sum of first three terms of an arithmetic series is 21. It is given that if the sum of the first two terms is subtracted from the third term then it would be 9.

We have to find the three terms of the series.

What is Arithmetic progression or sequence?

An arithmetic progression or sequence with common difference [tex]d[/tex] is given as -

a, a + d, a + 2d, a + 3d .....

According to the question -

a + (a + d) + (a + 2d) = 21

3a + 3d = 21

3(a + d)=21

a + d = 7                             (Eqn. 1)

(a + 2d) - (a + a + d) = 9

a + 2d - a - a - d = 9

d - a = 9

a = d - 9

Substituting the value of 'a' in Eqn. 1, we get -

d - 9 + d = 7

2d - 9 = 7

2d = 16

d = 8

Therefore -

a = 8 - 9

a = -1

The three terms are -

a = -1

a + d = -1 + 8 = 7

a + 2d = -1 + 2 x 8 = 15

Hence, the first three terms will be - -1, 7, 15

To solve more questions on Arithmetic Sequences and finding the terms, visit the link below -

https://brainly.com/question/20384906

#SPJ2

Rewrite using exponents

AxAxAxAxAxAxA



need answer quick!!! please

Answers

Answer:

A^7

Step-by-step explanation:

ABCD is a parallelogram, AE=16, AB=24, EC=2y-4, DE=67, and EB = 2x+7. Solve for x. (Round your answer to one decimal place, if necessary.) Plz help will give brainliest

Answers

Answer:

x = 30

Step-by-step explanation:

The diagonals are bisected so DE =EB

67 = 2x+7

Subtract 7

67-7 = 2x+7-7

60 =2x

Divide by 2

60/2 =2x/2

30 = x

Best answer gets brainliest and 5 stars

Answers

Answer:

A

Step-by-step explanation:

pgt states if the square of the length of the longest side of a triangle is equal to the sum of the squares of the other two sides, then the triangle is a right triangle.

Answer:

Does the answer help you?

30 Points cuz I need help ASAP

Answers

Answer:

the answer is option 1

Step-by-step explanation:

the negative angle => a quarter round angle (clockwise) = ¼ x ( -360°) = -90°

and

the positive angle => a full round angle + 270° = (360°)+270°

= 630°

Answer:

correct answer is -90 and 630

Step-by-step explanation:

will mark brainliest!!

Answers

Answer:

In March she sent 752 texts

In April she sent 617

Step-by-step explanation:

The phone bill is 35 + additional texts, so in both bills the value relative to the texts are:

72.6 - 35 = 37.6 for March

65.85 - 35 = 30.85 for April

Each texts costs 0.05, so if we divide the value relative to the texts by 0.05 we will have the number of texts she sent:

37.6/0.05 = 752 texts

30.85/0.05 = 617 texts

Other Questions
HELP ME PLZWhere in an argumentative essay would this excerpt most likely belong? I need help with 9-20 Use the correct form of the verb "ser" in the following sentences.Plz hurry I have to babysit soon. Thx En este mes, nuestro Per cumple 199 aos de independencia, estamos a un ao del bicentenario, por ello es muy importante refl exionar, tomar accin y comprometernos con el Per. Una forma es asumiendo nuestras responsabilidades frente al efecto invernadero. You invent of a new type of dog leash. You choose a market segmentation approach and decide to target the large national population of dog owners. After reviewing what identifies an ideal market you realize your segmentation approach does not meet any of the effective segmentation conditions. At this point you should: 2 6 + 3 * 4 2 + 7 * - 2 / Which of the following intervals is the graph decreasing? bi:combine each pair of sentences using enough...to V or enough for...to V1 the moon is very bright . we can play outdoor2 i have enough money . i can pay this bill3 my sister is old . she can driver a car Four times an angle is equal to half of its supplement. Find the measures of both angles. Find the place value of 8 in 2456.1387.TenthsHundredthsThousandthsThousands 1. The product of two consecutive numbers is 42. What are the numbers? Fill in the blanks with suitable preposition:1. It suddenly became so dark as the train went ________ the tunnel. 2. The restaurant is ________ the bank and the post office. 3. What language do you speak ________ English? 4. You are not going out ________ you finish your homework 5. The book was written ________ Mark Twain. 6. I will show you the picture ________ the palace. 7. The TV is ________ the corner of the room. 8. Many of us eat ________ fork and spoon. 9. There is a good restaurant ________ the end of the street. 10. He is fond ________ TV. 11. The cat jumped ________ the fence. 12. He has a positive attitude ________ his work. 13. The artist dipped his brush ________ the bright colours. 14. What do you think ________ the concert? Find X in the picture Norma gives her elderly mother a ride to the grocery store once a week when she does her own grocery shopping. On a form at work, she lists this as a community service and hides the fact that it's her own mother she's helping because she knows her boss values volunteerism and actively encourages all employees to contribute to the community. When her boss praises her for her commitment to community service, Norma feels pleased and optimistic that she will soon get a raise. According to virtue-based ethics, why is Norma's action unethical?a. She is acting out of a sense of compassion and goodwill. b. She is prioritizing monetary gain over honesty. c. Lying is always wrong. d. She is not cultivating any virtues by helping her mother. Who said the following, and what does it mean? Conceit, more rich in matter than in words, Brags of his substance, not of ornament. They are but beggars that can count their worth; But my true love is grown to such excess I cannot sum up half my sum of wealth. (II. vi. 33-37) Juliet; she is saying that she doesn't care about money. Lady Capulet; she is reminding Juliet how lucky she is to be marrying Paris. Romeo; he is saying that true understanding has made him realize how very lucky he is to be marrying Juliet. Juliet; she is saying that true understanding is enriched by reality and worth more than outward appearances. PLS HELP ME WITH MY PYTHON HW Which of these numbers are greater than 24? Check all that apply.O A. 12B. 15O C. 42D. 41E. 13D F. 18 We don't need society to survive. Statements. Health Hi, I need help. Im supposed to say why we dont need society to survive health in statements (2x+1)(x-4)(6x-5)(3x+2) plzzzzzzzz helllllllppppppppp Why do you think more people didnt take the risks Gies took to help?